elina.ayzenberg
Thanks Received: 0
Forum Guests
 
Posts: 2
Joined: May 10th, 2011
 
 
 

Q12 - It is primarily by raising

by elina.ayzenberg Mon May 30, 2011 6:20 pm

Hi,

I am a bit confused on how the statement "It is primarily by raising interest rates that central bankers curb inflation" is a conclusion, and how the following statement is a premise for it. If you could explain this it would be a great help.

Thanks!
 
giladedelman
Thanks Received: 833
LSAT Geek
 
Posts: 619
Joined: April 04th, 2010
 
 
 

Re: Q12 - It is primarily by raising

by giladedelman Tue May 31, 2011 4:35 pm

It's not! Check your answer key! The answer to this one is (C): it's a premise in support of the conclusion introduced in the last line. ("Thus, central bankers' success ...") Everything leading up to the last line is providing support for that conclusion.
User avatar
 
maryadkins
Thanks Received: 640
Atticus Finch
Atticus Finch
 
Posts: 1261
Joined: March 23rd, 2011
 
This post thanked 2 times.
 
 

Re: Q12 - It is primarily by raising interest rates that central

by maryadkins Fri Jul 15, 2011 3:42 pm

I took a swing at this one :)

12. (C)
Question Type: Analyze Argument Structure (Determine Function of a Component)

Bankers mostly curb inflation by raising interest rates, we are told. But it takes two years for this to work, and it happens before inflation ever gets too bad. As a result, people never see the bad inflation and don’t realize it worked"”they just get mad about the high interest rates! (An analogy is increased security measures to prevent terrorism that anger people"”they may hate the measures and complain they aren’t necessary, but perhaps they aren’t seeing the harm that’s being prevented because it’s being prevented.) The conclusion, after "thus," is that bankers’ success at solving inflation problems now can end up making it harder for them in the long-term. The core is:

Bankers raise interest rates to curb inflation before it gets too bad

+

People don’t see inflation, perceive the raised interest rates as needless ->

Raising interest rates to prevent inflation makes people more likely to get mad about it later (because they don’t see why it’s necessary)

We are looking for the role of the underlined text, above. We can rule it out as the conclusion, right away. The conclusion is the last sentence (after "thus" _ flag word!). So right away we know that it is either background information or a premise in a series of complementary premises. The phrase is offered as one of the premises to support the conclusion"”bankers are raising rates to curb inflation, and when they do it well, it causes an unwanted effect.

(A) is incorrect. It is far from a complete explanation.
(B) is incorrect. The claim that it takes 2 years to take effect is not offered as an explanation for why bankers raise interest rates to curb inflation. Also, the statement about it taking 2 years is not a claim"”it’s just a premise. We take it as fact.
(D) is incorrect. Once we identify the parts of the argument, we can knock out this answer after a quick scan because the relevant text is not the conclusion.
(E) is incorrect, but it sure looking tempting at first! When we compare (E) to (C), however, we see that the only difference is how they describe the conclusion. (C) accurately describes the conclusion. (E) doesn’t give the conclusion"”it states a premise. Knock it out!
 
peg_city
Thanks Received: 3
Forum Guests
 
Posts: 152
Joined: January 31st, 2011
Location: Winnipeg
 
 
trophy
First Responder
 

Re: Q12 - It is primarily by raising interest rates that central

by peg_city Mon Apr 02, 2012 3:29 pm

The problem I have with this one is that 'unless inflation is readily apparent, interest rate hikes generally will be perceived as needlessly restraining a growing economy' is a subsidiary conclusion, not the final conclusion. Thus, the first sentence is a premise to the subsidiary conclusion.

What did I do wrong?

Thanks
 
timmydoeslsat
Thanks Received: 887
Atticus Finch
Atticus Finch
 
Posts: 1136
Joined: June 20th, 2011
 
 
trophy
Most Thanked
trophy
First Responder
 

Re: Q12 - It is primarily by raising interest rates that central

by timmydoeslsat Tue Apr 03, 2012 12:18 pm

peg_city Wrote:The problem I have with this one is that 'unless inflation is readily apparent, interest rate hikes generally will be perceived as needlessly restraining a growing economy' is a subsidiary conclusion, not the final conclusion. Thus, the first sentence is a premise to the subsidiary conclusion.

What did I do wrong?

Thanks

That is not a subsidiary conclusion. It is another premise. It is often the case that transition language is a marker of a conclusion, but in this case, it is not. Same with the second line of the stimulus stating "but an increase in interest rates takes up to two years." We are not attempting to prove that claim, it is simply another premise, as is the sentence you brought up.

Transitioning language is a huge indicator of a conclusion, but we must be sure that other things are supporting it. The statement "unless inflation is readily apparent, interest rate hikes generally will be perceived as needlessly restraining a growing economy" does not receive support from the first line.
 
minhtientm249
Thanks Received: 0
Forum Guests
 
Posts: 22
Joined: February 29th, 2012
 
 
 

Re: Q12 - It is primarily by raising

by minhtientm249 Sat Jan 05, 2013 10:32 pm

I've come to realize that "it is primarily by raising interest rate....curb inflation" is either background information or a premise. But how come it's not background information? I think it makes sense if that statement is a background information. Could you please explain why it's a premise and not a background information?

Thank you.
User avatar
 
ttunden
Thanks Received: 0
Atticus Finch
Atticus Finch
 
Posts: 146
Joined: August 09th, 2012
 
 
 

Re: Q12 - It is primarily by raising

by ttunden Mon Aug 11, 2014 2:00 am

I too had the same prephase where I thought it would be either background or a premise

I can safely eliminate A,B and D. I was down to C and E. So, are you telling me that E is wrong because the "unless inflation apparent..." is not the conclusion? and not a subsidiary conclusion?

It looks like that statement can support both of the posited conclusion in C and E. That's why I was stuck on this question during the test and blind review.

The irony is, is that I correctly identified the main conclusion of this argument but I wasn't sure if that middle part was a subsidiary conclusion. If there was a subsidiary conclusion, does the correct answer have to say supports subsidiary conclusion, or can it get away with just "conclusion"? Anyways to reiterate, is E only wrong because it misidentifies the conclusion of the argument? is that all?
User avatar
 
maryadkins
Thanks Received: 640
Atticus Finch
Atticus Finch
 
Posts: 1261
Joined: March 23rd, 2011
 
 
 

Re: Q12 - It is primarily by raising

by maryadkins Sat Aug 16, 2014 12:40 pm

Yes, that is why (E) is wrong and yes, if it says "conclusion" it needs to be the main conclusion not a subsidiary conclusion.